3
$\begingroup$

Consider the group $\mathbb{Z}_2^n$, equipped it with the following dot product: for $a=(a_1,\dots,a_n)\in \mathbb{Z}_2^n$ and $b=(b_1,\dots,b_n)\in \mathbb{Z}_2^n$, define $a\cdot b :=a_1b_1+\dots+a_nb_n.$

Let $A \subseteq \mathbb{Z}_2^n$ be a set of cadinality $a$, where $2^{n-1}<a<2^n$. Define $$r(A) := |\{(x, y, z) \in A^3 : x + y = z\}|.$$ Using basic Fourier analysis, one can show that $$r(A) = \frac{1}{2^n} \sum_{\chi \in \widehat{\mathbb{Z}_2^n}} \sum_{x, y, z \in A} \chi(x + y + z).$$ Since the characters of $\widehat{\mathbb{Z}_2^n}$ are well known, we can rewrite this as $$r(A)=\frac{1}{2^n}\sum_{\varepsilon\in \{0,1\}^n}\sum_{x,y,z\in A}(-1)^{\varepsilon\cdot (x+y+z)}.$$

Define $S(A):=\sum\limits_{\varepsilon\in \{0,1\}^n}\sum\limits_{x,y,z\in A}(-1)^{\varepsilon\cdot (x+y+z)}$. I would like to prove the following lower bound for $S(A)$: $$S(A)\geq 2^n\Big(2^n-2^{1+\lfloor \log_2(2^n-a)\rfloor}\Big)\Big(3a-2^{n+1}+2^{1+\lfloor \log_2(2^n-a)\rfloor}\Big).$$

I currently have no idea how to prove this, but perhaps someone else does. Thank you very much!

$\endgroup$
6
  • 3
    $\begingroup$ If $A = \{w \in \mathbb{Z}_2^n : w_1 = 1\}$, then $r(A) = 0$. $\endgroup$ Commented Apr 24 at 9:57
  • $\begingroup$ @mathworker21, Indeed, this is a nice observation. If we denote the whole sum by $S(A)$, we obtain: $S(A) = 2^n r(A) - a^3 \geq -a^3$. You've shown that the bound is sharp when $r(A) = 0$, which is good. However, I’m particularly interested in the case when $2^{n-1} + 1 \leq a < 2^n$. $\endgroup$ Commented Apr 24 at 12:30
  • 1
    $\begingroup$ We have the trivial bounds $\lvert A\rvert(2\lvert A\rvert-2^n)\leq r(A)\leq \lvert A\rvert^2$. In particular if $\lvert A\rvert/2^n=\alpha\in (1/2,1)$ then $\alpha(2\alpha-1)\leq r(A)2^{-2n}\leq \alpha^2$. $\endgroup$ Commented Apr 29 at 10:42
  • $\begingroup$ Can you explain where your particular lower bound for $S(A)$ comes from? Why do you believe it is true? $\endgroup$ Commented Apr 29 at 10:43
  • 1
    $\begingroup$ @ThomasBloom, The lower for $S(A)$ comes from the following fact: the expression $\Big(2^n-2^{1+\lfloor \log_2(2^n-a)\rfloor}\Big)\Big(3a-2^{n+1}+2^{1+\lfloor \log_2(2^n-a)\rfloor}\Big)$ is the value of $r(B_a)$, where $B_a$ corresponds to the binary expansions of the integers in $\{2^n-a,\dots,2^n-1\}$. Taking this into account, this should (I believe!) give an optimal lower bound for $S(A)$. $\endgroup$ Commented Apr 29 at 13:07

1 Answer 1

0
$\begingroup$

$\newcommand{\Z}{\mathbb{Z}}$ You ask an interesting question. Denote by $1_A \colon \Z_2^n \to \{0, 1\}$ the indicator function of $A$. Let $\widehat{1_A}$ denote its Fourier-Hadamard transform, given by $\widehat{1_A}(a)=\sum_{x \in \Z_2^n}(-1)^{x \cdot a} 1_A(x)$. An alternate representation of $r(A)$ is $r(A) = \frac{1}{2^n} \sum_{x \in \Z_2^n} (\widehat{1_A})^3(x)$.

If $r(A)=0$ then $A$ is sum-free, i.e. $A \cap (A+A)=\emptyset$, but the maximal size of a sum-free set in $\Z_2^n$ is $2^{n-1}$. So, when $|A| >2^{n-1}$ (as you are interested in), we always have $r(A) >0$.

Let $A'$ be the set $\{1\} \times \Z_2^{n-1}$. Then $A'$ is sum-free of size $2^{n-1}$. Also $|\{ (x,y) \in (A')^2 : x+y=u| = 2^{n-1}$ for all nonzero $u\in \{0\} \times \Z_2^{n-1}$.

Let $A = A' \cup \{u\}$ for some nonzero $u\in \{0\} \times \Z_2^{n-1}$. Then $r(A) = |\{ (x,y,z) \in A^3 : x+y=z, u \in \{ x,y,z\}\}| = 3|\{ (x,y) \in (A')^2 : x+y=u\}= 3 \cdot 2^{n-1}$.

It seems plausible that this is minimal for all $A$ such that $|A| \geq 2^{n-1}+1$.

Edit: To find a lower bound, it suffices to consider when $|A| =2^{n-1}+1$ as $r(A) \geq r(B)$ when $A \supset B$. So, assume $A \subseteq \Z_2^n$ is a subset of size $2^{n-1}+1$. Moreover, without loss of generality, assume $0 \notin A$ because if $0\in A$, then $$r(A) = |\{(x,y,z) \in A^3 : x+y+z=0\}| \geq 3(|A|-1) = 3\cdot 2^{n-1}$$ which we already have achieved via the construction above. If $A'=A \setminus \{u\}$ for some $u \in A$, then $$\begin{aligned} 2^nr(A) &= \sum_{x \in \Z_2^n} (\widehat{1_A})^3(x) \\ &= \sum_{x \in \Z_2^n} \left( (\widehat{1_{A'}})^3(x) + (-1)^{u \cdot x} + 3(-1)^{u \cdot x} (\widehat{1_{A'}})^2(x) + 3(\widehat{1_{A'}})(x) \right)\\ &= \sum_{x \in \Z_2^n} (\widehat{1_{A'}})^3(x) + 0 + 3\widehat{(\widehat{1_{A'}})^2}(u) + 3 \cdot 2^n 1_{A'}(0) \\ &= 2^n r(A') + 3 \cdot 2^n |A' \cap (u+A')|. \end{aligned}$$ So, $$r(A) \geq \min_{\substack{A' \subseteq \Z_2^n \setminus \{0\}, |A'|=2^{n-1}\\ u \in \Z_2^n \setminus (A' \cup \{0\})}} (r(A')+3 |A' \cap (u+A')|).$$

$\endgroup$
3
  • $\begingroup$ Thanks for your reply! Just a small comment: I did not have a chance to read your answer in detail but my goal is to minimize $r(A)$ over all $A\subseteq\mathbb{Z}_2^n$ such that $|A|=a$ where $a$ is fixed within the range $2^{n-1}<a<2^n$ $\endgroup$ Commented Apr 25 at 5:04
  • $\begingroup$ Hi! My answer is incomplete and does not provide bounds in terms of $|A|$. It focuses on a minimum of $r(A)$ for all $A$ with $2^{n-1}<|A| < 2^n$ which seems like it should be $3 \cdot 2^{n-1}$ but seems nontrivial to prove. $\endgroup$ Commented Apr 25 at 14:21
  • $\begingroup$ Also, here's a thought. Suppose $A$ does not include $0$, and let $M_A$ be the matrix whose columns are the vectors in $A$. The nullspace of $M_A$ is a linear code, and I believe the problem is equivalent to bounding the number of code words of weight $3$. $\endgroup$ Commented Apr 25 at 14:25

You must log in to answer this question.

Start asking to get answers

Find the answer to your question by asking.

Ask question

Explore related questions

See similar questions with these tags.